Biochem for CAS #3

¡Supera tus tareas y exámenes ahora con Quizwiz!

Routine examination of the urine of an asymptomatic pediatric patient showed a positive reaction with Clinitest (a copper reduction method of detecting reducing sugars), but a negative reaction with the glucose oxidase test. Which one of the following sugars is least likely to be present (assuming a single elevated saccharide)? A. Lactose B. Fructose C. Sucrose D. Xylulose E. Galactose

Correct answer = C. Clinitest is a nonspecific test that produces a change in color if urine is positive for reducing substances, including reducing sugars (glucose, fructose, galactose, xylulose, lactose), amino acids, ascorbic acid, and certain drugs and drug metabolites. Because sucrose is not a reducing sugar, it is not detected by Clinitest. Glucose oxidase method will not detect increased levels of galactose or other sugars in urine. It is therefore important that a copper reduction method be used as a screening test. In those instances when the copper method is positive and the glucose oxidase method is negative, glucosuria is ruled out.

A young black man entered his physician's office complaining of bloating and diarrhea. His eyes were sunken and the physician noted additional signs of dehydration. The patient's temperature was normal. He explained that the episode had occurred following a birthday party at which he had participated in an ice cream eating contest. The patient reported prior episodes of a similar nature following ingestion of a significant amount of dairy products. This clinical picture is most probably due to a deficiency in: A.salivary α-amylase. B.isomaltase. C.pancreatic α-amylase. D.sucrase. E.lactase.

Correct answer = E. The physical symptoms suggest a deficiency in an enzyme responsible for carbohydrate degradation. The symptoms observed following the ingestion of dairy products suggest that the patient is deficient in lactase.

A patient was diagnosed with mtDNA mutation that led to reduced complex I activity. This patient would have difficulties in which of the following electron transfers? A. Succinate to complex III B. Cytochrome c to complex IV C. Coenzyme Q to complex III D. NADH to Coenzyme Q E. CoQ to oxygen

D.

A pair of farm workers in Mexico was spraying pesticides on crops when both developed the following sever symptoms: heavy labored breathing and significantly elevated temperature. The pesticides contained an agent that interfered with OxPhos. which most closely resembles which of the following known inhibitors? A. Oligomycin B. Amytal C. Cyanide D. Rotenone E. Dinitroplenol

E. DNP (2,4-Dinitrophenol) disrupts the proton gradient.

Which of the following has the strongest tendency to gain electrons? A. CoQ B. Cytochrome C C. FAD D. NAD E. O2

E. O2 is the final electron acceptor.

A 25-year-old woman presents with chronic fatigue. A series of blood tests is ordered, and the results suggest that her red blood cell count is low because of iron deficiency anemia. Such a deficiency would lead to fatigue because of which of the following? A. Her decrease in Fe-S centers is impairing the transfer of electrons in the electron-transport chain. B. She is not producing enough H2O in the electrontransport chain, leading to dehydration, which has resulted in fatigue. C. Iron forms a chelate with NADH and FAD(2H) that is necessary for them to donate their electrons to the electron-transport chain. D. Iron acts as a cofactor for alpha-ketoglutarate DH in the TCA cycle, a reaction required for the flow of electrons through the electron-transport chain. E. Iron accompanies the protons that are pumped from the mitochondrial matrix to the cytosolic side of the inner mitochondrial membrane. Without iron, the proton gradient cannot be maintained to produce adequate ATP.

The answer is A. A deficiency of Fe-S centers in the electron-transport chain would impair the transfer of electrons down the chain and reduce ATP production by oxidative phosphorylation. Answer B is incorrect because the decreased production of water from the electron-transport chain is not of sufficient magnitude to cause her to become dehydrated. Answer C is incorrect because iron does not form a chelate with NADH and FAD(2H). Answer D is incorrect because iron is not a cofactor for alpha-ketoglutarate dehydrogenase. Answer E is incorrect because iron does not accompany the protons that make up the proton gradient.

In a glucose tolerance test, an individual in the basal metabolic state ingests a large amount of glucose. If the individual is normal, this ingestion should result in which of the following? A. An enhanced glycogen synthase activity in the liver B. An increased ratio of glycogen phosphorylase a to glycogen phosphorylase b in the liver C. An increased rate of lactate formation by red blood cells D. An inhibition of PP-1 activity in the liver E. An increase of cAMP levels in the liver

The answer is A. After ingestion of glucose, insulin levels rise, cAMP levels within the cell drop (thus, E is incorrect), and protein-phosphatase-I is activated (thus, D is incorrect). Glycogen phosphorylase a is converted to glycogen phosphorylase b by the phosphatase (thus, B is incorrect), and glycogen synthase is activated by the phosphatase. Red blood cells continue to use glucose at their normal rate, thus lactate formation will remain the same (thus, C is incorrect).

The apoproteins B-48 and B-100 are similar with respect to which of the following? A. They are synthesized from the same gene. B. They are derived by alternative spicing of the same hnRNA. C. apoB-48 is a proteolytic product of apoB-100. D. Both are found in mature chylomicrons. E. Both are found in very low-density lipoproteins.

The answer is A. Both apoB-48 and apoB-100 are derived from the same gene and from the same mRNA (there is no difference in splicing between the two, thus B is incorrect). However, RNA editing introduces a stop codon in the message such that B-48 stops protein synthesis approximately 48% along the message. Thus, proteolytic cleavage is not correct. B-48 is found only in chylomicrons, and B-100 is found only in VLDL particles.

The oxidation of fatty acids is best described by which of the following sets of reactions? A. Oxidation, hydration, oxidation, carbon-carbon bond breaking B. Oxidation, dehydration, oxidation, carbon-carbon bond breaking C. Oxidation, hydration, reduction, carbon-carbon bond breaking D. Oxidation, dehydration, reduction, oxidation, carbon- carbon bond breaking E. Reduction, hydration, oxidation, carbon-carbon bond breaking

The answer is A. Fatty acid oxidation is initiated by the acyl-CoA dehydrogenase (an oxidation step), followed by hydration of the double bond formed in the first step, followed by the hydroxyacyl-CoA dehydrogenase step (another oxidation), and then attack of the Beta-carbonyl by CoA, breaking a carbon-carbon bond (the thiolase step).

Which of the following is involved in the synthesis of triacylglycerols in adipose tissue? A. Fatty acids obtained from chylomicrons and VLDL B. Glycerol 3-phosphate derived from blood glycerol C. 2-Monoacylglycerol as an obligatory intermediate D. LPL to catalyze the formation of ester bonds E. Acetoacetyl-CoA as an obligatory intermediate

The answer is A. Fatty acids, cleaved from the triacylglycerols of blood lipoproteins by the action of lipoprotein lipase, are taken up by adipose cells and react with coenzyme A to form fatty acyl-CoA. Glucose is converted via dihydroxyacetone phosphate to glycerol 3-phosphate, which reacts with fatty acyl-CoA to form phosphatidic acid (adipose tissue lacks glycerol kinase, so it cannot use glycerol directly). After inorganic phosphate is released from phosphatidic acid, the resulting diacylglycerol reacts with another fatty acyl-CoA to form a triacylglycerol, which is stored in adipose cells (2-monoacylglycerol is an intermediate of triglyceride synthesis only in the intestine, not in adipose tissue).

Succinate dehydrogenase differs from all other enzymes in the TCA cycle in that it is the only enzyme that displays which of the following characteristics? A. It is embedded in the inner mitochondrial membrane. B. It is inhibited by NADH. C. It contains bound FAD. D. It contains Fe-S centers. E. It is regulated by a kinase.

The answer is A. Succinate dehydrogenase is the only TCA cycle enzyme located in the inner mitochondrial membrane. The other enzymes are in the mitochondrial matrix. Answer B is incorrect because succinate dehydrogenase is not regulated by NADH. Answer C is incorrect because alpha-ketoglutarate dehydrogenase also contains a bound FAD (the difference is that the FAD[2H] in alpha-ketoglutarate dehydrogenase donates its electrons to NAD+, whereas the FAD[2H] in succinate dehydrogenase donates its electrons directly to the electron-transfer chain). Answer D is incorrect because both succinate dehydrogenase and aconitase have Fe-S centers. Answer E is incorrect because succinate dehydrogenase is not regulated by a kinase. Kinases regulate enzymes by phosphorylation (e.g., the regulation of pyruvate dehydrogenase occurs through reversible phosphorylation).

Certain patients with abetalipoproteinemia frequently have difficulty maintaining blood volume; their blood has trouble clotting. This symptom is attributable to which of the following? A. Inability to produce chylomicrons B. Inability to produce VLDL C. Inability to synthesize clotting factors D. Inability to synthesize fatty acids E. Inability to absorb short-chain fatty acids

The answer is A. The clotting problems are caused by a lack of vitamin K, a lipid-soluble vitamin. Vitamin K is absorbed from the diet in mixed micelles and packaged with chylomicrons for delivery to the other tissues. Individuals with abetalipoproteinemia lack the microsomal triglyceride transfer protein and cannot produce chylomicrons effectively, thus vitamin K deficiency can result. Such patients also cannot produce VLDL, but lipidsoluble vitamin distribution does not depend on VLDL particles, only on chylomicrons. The other answers are all incorrect statements.

Which one of the following apoproteins acts as a cofactor activator of the enzyme lipoprotein lipase (LPL)? A. ApoCIII B. ApoCII C. ApoB100 D. ApoB48s E. ApoE

The answer is B. ApoCIII appears to inhibit the activation of LPL. ApoE acts as a ligand in binding several lipoproteins to the LDL receptor, the LDL receptor-related protein (LRP), and possibly to a separate apoE receptor. ApoB48 is required for the normal assembly and secretion of chylomicrons from the small bowel, whereas apoB100 is required in the liver for the assembly and secretion of VLDL. ApoCII is the activator of LPL.

Which one of the following sequences places the lipoproteins in the order of most dense to least dense? A. HDL/VLDL/chylomicrons/LDL B. HDL/LDL/VLDL/chylomicrons C. LDL/chylomicrons/HDL/VLDL D. VLDL/chylomicrons/LDL/HDL E. LDL/chylomicrons/VLDL/HDL

The answer is B. Because chylomicrons contain the most triacylglycerol, they are the least dense of the blood lipoproteins. Because VLDL contains more protein, it is denser than chylomicrons. Because LDL is produced by degradation of the triacylglycerol in VLDL, LDL is denser than VLDL. HDL is the densest of the blood lipoproteins. It has the most protein and the least triacylglycerol.

The most abundant component of chylomicrons is which of the following? A. apoB-48 B. Triglyceride C. Phospholipid D. Cholesterol E. Cholesterol ester

The answer is B. Chylomicrons transport dietary lipids, and >80% of the chylomicron is triglyceride. All other components are present at <10%, hence all other answers are incorrect.

Which of the following medications, given chronically, could create a physiologic response that would be a major source of free-radical production? (A) Ciprofloxacin (B) Isoniazide (C) Cimetidine (D) Ketaconazole (E) None of these drugs has the potential to increase free-radical formation.

The answer is B. Ciprofloxacin, cimetidine, and ketaconazole all inhibit cytochrome P450. Their mode of detoxification does not require the actions of cytochrome P450. Isoniazide, however, induces cytochrome P450 formation as a means of oxidizing the drug to remove it from the body. Cytochrome P450 enzymes are a major source of free-radical production that can occur when electrons are accidentally leaked from reactions and react with molecular oxygen.

Assume that an individual has been eating excess calories daily such that he will gain weight. Under which of the following conditions will the person gain weight most rapidly? A. If all the excess calories are due to carbohydrate. B. If all the excess calories are due to triacylgycerol. C. If all the excess calories are split 50%/50% between carbohydrate and triacylgycerol. D. If all the excess calories are split 25%/75% between carbohydrate and triacylgycerol. E. It makes no difference what form the excess calories are in.

The answer is B. Consider the energy required to convert dietary carbohydrates to triacylglycerol. Some ATP is generated from glycolysis and the pyruvate dehydrogenase reaction, but energy is also lost as the fatty acids are synthesized (the synthesis of each malonyl-CoA requires ATP, and the reduction steps require two molecules of NADPH). Dietary fat, however, only requires activation and attachment to glycerol; the fatty acid chain does not need to be synthesized. Therefore, it requires less energy to package dietary fat into chylomicrons than it does to convert dietary carbohydrate into fatty acids for incorporation into VLDL. Thus, weight gain will be more rapid if all the excess calories are derived from fat as opposed to carbohydrates.

A patient with hyperlipoproteinemia would be most likely to benefit from a low-carbohydrate diet if the lipoproteins that are elevated in blood are which of the following? A. Chylomicrons B. VLDL C. HDL D. LDL E. IDL

The answer is B. Dietary carbohydrate is converted to lipid in the liver and exported via VLDL. Thus, a lowcarbohydrate diet will reduce VLDL formation and reduce the hyperlipoproteinemia.

Consider the following experiment. Carefully isolated liver mitochondria are incubated in the presence of a limiting amount of malate. Three minutes after adding the substrate, cyanide is added, and the reaction is allowed to proceed for another 7 minutes. At this point, which of the following components of the electron-transfer chain will be in an oxidized state? A. Complex I B. Complex II C. Complex III D. Coenzyme Q E. Cytochrome C

The answer is B. For a component to be in the oxidized state, it must have donated, or never received, electrons. Complex II will metabolize succinate to produce fumarate (generating FAD[2H]), but no succinate is available in this experiment. Thus, complex II never sees any electrons and is always in an oxidized state. The substrate malate is oxidized to oxaloacetate, generating NADH, which donates electrons to complex I of the electron-transport chain. These electrons are transferred to coenzyme Q, which donates electrons to complex III, to cytochrome c, and then to complex IV. Cyanide will block the transfer of electrons from complex IV to oxygen, so all previous complexes containing electrons will be backed up and the electrons will be "stuck" in the complexes, making these components reduced. Thus, answers A and C through E must be incorrect.

Intravenous fructose feeding can lead to lactic acidosis caused by which of the following? A. Bypassing the regulated pyruvate kinase step B. Bypassing the regulated phosphofructokinase-1 (PFK-1) step C. Allosterically activating aldolase B D. Allosterically activating lactate dehydrogenase E. Increasing the [ATP]:[ADP] ratio in liver

The answer is B. Fructose is converted to fructose 1-phosphate by fructokinase, and aldolase B in the liver splits the fructose 1-P into glyceraldehyde and dihydroxyacetone phosphate. Thus, the major regulated step of glycolysis, PFK-1, is bypassed and PEP is rapidly produced. As the [PEP] increases, pyruvate kinase produces pyruvate. As the glyceraldehyde-3-phosphate dehydrogenase reaction is proceeding rapidly (remember that fructokinase is a high Vmax enzyme, so there is a lot of substrate proceeding through the glycolytic pathway), the intracellular [NADH]/[NAD+] ratio is high, and the pyruvate produced is converted to lactate in order to regenerate NAD+. Thus, the pyruvate kinase step is not bypassed (thus, A is incorrect). Neither aldolase B nor lactate dehydrogenase is allosterically regulated (thus, C and D are incorrect), and even though the [ATP]:[ADP] ratio is high in the liver under these conditions, the ratio does not affect lactate formation (thus, E is incorrect).

The degradation of glycogen normally produces which of the following? A. More glucose than glucose 1-phosphate B. More glucose 1-phosphate than glucose C. Equal amounts of glucose and glucose 1-phosphate D. Neither glucose or glucose 1-phosphate E. Only glucose 1-phosphate

The answer is B. Glycogen phosphorylase produces glucose 1-phosphate; the debranching enzyme hydrolyzes branch points and thus releases free glucose. Ninety percent of the glycogen contains alpha-1,4-bonds and only 10% are alpha-1,6-bonds, so more glucose 1-phosphate will be produced than glucose.

A chronic alcoholic has been admitted to the hospital because of a severe hypoglycemic episode brought about by excessive alcohol consumption for the past 5 days. A blood lipid analysis indicates much higher than expected VLDL levels. The elevated VLDL is attributable to which of the following underlying cause? A. Alcohol-induced inhibition of lipoprotein lipase B. Elevated NADH levels in the liver C. Alcohol-induced transcription of the apo B-100 gene D. NADH activation of phosphoenolpyruvate carboxykinase E. Acetaldehyde induction of enzymes on the endoplasmic reticulum

The answer is B. Metabolism of ethanol leads to production of NADH in the liver, which will inhibit fatty acid oxidation in the liver. Because the patient has not eaten for 5 days, the insulin/glucagon ratio is low, hormone-sensitive lipase is activated, and fatty acids are being released by the adipocyte and taken up by the muscle and liver. However, because the liver NADH levels are high as a result of ethanol metabolism, the fatty acids received from the adipocyte are repackaged into triacylglycerol (the high NADH promotes the conversion of dihydroxyacetone phosphate to glycerol 3-phosphate as well) and secreted from the liver in the form of VLDL. None of the other answers is a correct statement. `

Which of the following would be expected for a patient with an OXPHOS disease? A. A high ATP:ADP ratio in the mitochondria B. A high NADH:NAD+ ratio in the mitochondria C. A deletion on the X chromosome D. A high activity of complex II of the electrontransport chain E. A defect in the integrity of the inner mitochondrial membrane

The answer is B. NADH would not be reoxidized as efficiently by the electron-transport chain, and the NADH/ NAD+ ratio would increase. Answer A is incorrect because ATP would not be produced at a high rate. Therefore, ADP would build up, and the ATP:ADP ratio would be low. Answer C is incorrect because OXPHOS diseases can be caused by mutations in nuclear or mitochondrial DNA, and not all OXPHOS proteins are encoded by the X chromosome. Answer D is incorrect because, depending on the nature of the mutation, the activity of complex II of the electron-transport chain might be normal or decreased, but there is no reason to expect increased activity. Answer E is incorrect because the integrity of the inner mitochondrial membrane would not necessarily be affected. It could be, but it would not be expected for all patients with OXPHOS disorders.

The polyol pathway of sorbitol production and the hexose monophosphate (HMP) shunt pathway are linked by which of the following? A. The HMP shunt produces 6-phosphogluconate, an intermediate in the polyol pathway. B. The HMP shunt produces NADPH, which is required for the polyol pathway. C. The HMP shunt produces ribitol, an intermediate of the polyol pathway. D. Both pathways use glucose as the starting material. E. Both pathways use fructose as the starting material.

The answer is B. Reduction of sugar aldehydes to alcohols requires NADPH, which is generated primarily by the pentose phosphate pathway. 6-Phosphogluconate is not a polyol (thus, A is incorrect; 6-phosphogluconate is glucose oxidized at position 1 to form a carboxylic acid); ribitol is not a product of the pentose phosphate pathway (thus, B is incorrect; ribulose 5-phosphate is a product of the pentose phosphate pathway); the HMP shunt uses glucose 6-phosphate as the starting material, not free glucose as in the sorbitol pathway (thus, D and E are incorrect).

A 20-year-old woman with diabetes mellitus was admitted to the hospital in a semiconscious state with fever, nausea, and vomiting. Her breath smelled of acetone. A urine sample was strongly positive for ketone bodies. Which one of the following statements about this woman is correct? A. A blood glucose test will probably show that her blood glucose level is much lower than 80 mg/dL. B. An injection of insulin will decrease her ketone body production. C. She should be given a glucose infusion so she will regain consciousness. D. Glucagon should be administered to stimulate glycogenolysis and gluconeogenesis in the liver. E. The acetone was produced by decarboxylation of the ketone body !-hydroxybutyrate.

The answer is B. The acetone on the woman's breath (which is produced by decarboxylation of acetoacetate; thus, E is incorrect) and the ketones in her urine indicate that she is in diabetic ketoacidosis. This is caused by low insulin levels, so her blood glucose levels are high because the glucose is not being taken up by the peripheral tissues (thus, A and C are incorrect). An insulin injection will reduce her blood glucose levels and decrease the release of fatty acids from adipose triglycerides. Consequently, ketone body production will decrease. Glucagon injections would just exacerbate the woman's current condition (thus, D is incorrect).

Bile salts must reach a particular concentration within the intestinal lumen before they are effective agents for lipid digestion. This is because of which of the following? A. The bile salt concentration must be equal to the triglyceride concentration. B. The bile salt solubility in the lumen is a critical factor. C. The ability of bile salts to bind lipase is concentration dependent. D. The bile salts cannot be reabsorbed in the ileum until they reach a certain concentration. E. The bile salts do not activate lipase until they reach a particular concentration.

The answer is B. The bile salts must be above their critical micelle concentration to form micelles with the components of lipase digestion, fatty acids, and 2-monoacylgycerol. In the absence of micelle formation, lipid absorption would not occur. The critical micelle concentration is independent of triglyceride concentration (thus, A is incorrect). Bile salts do not bind or activate lipase (thus, C and E are incorrect). The absorption of bile salts in the ileum is not related to digestion (thus, D is i ncorrect).

A major role of glycolysis is which of the following? A. To synthesize glucose B. To generate energy C. To produce FAD(2H) D. To synthesize glycogen E. To use ATP to generate heat

The answer is B. The major roles of glycolysis are to generate energy and to produce precursors for other biosynthetic pathways. Gluconeogenesis is the pathway that generates glucose (thus, A is incorrect), FAD(2H) is produced in the mitochondria by a variety of reactions but not glycolysis (thus, C is incorrect), glycogen synthesis occurs under conditions in which glycolysis is inhibited (thus, D is incorrect), and glycolysis does not hydrolyze ATP to generate heat (i.e., nonshivering thermogenesis; thus, E is incorrect).

Which of the following statements correctly describes an aspect of glycolysis? A. ATP is formed by oxidative phosphorylation. B. Two molecules of ATP are used in the beginning of the pathway. C. Pyruvate kinase is the rate-limiting enzyme. D. One molecule of pyruvate and three molecules of CO2 are formed from the oxidation of one glucose molecule. E. The reactions take place in the matrix of the mitochondria.

The answer is B. The pathway consumes 2 ATP at the beginning of the pathway and produces 4 ATP at the end of the pathway for each molecule of glucose. Therefore, the net energy production is 2 ATP for each molecule of glucose. Glycolysis synthesizes ATP via substrate-level phosphorylation, not oxidative phosphorylation (thus, A is incorrect) and synthesizes two molecules of pyruvate in the process (thus, D is incorrect). The pathway is cytosolic (thus, D is incorrect), and the rate-limiting step is the one catalyzed by PFK-1 (thus, C is incorrect)

Which of the following steps in the biosynthesis of cholesterol is the committed rate-limiting step? A. The condensation of acetoacetyl-CoA with a molecule of acetyl-CoA to yield beta-hydroxy-beta-methylglutarylCoA (HMG-CoA) B. The reduction of HMG-CoA to mevalonate C. The conversion of mevalonate to two activated isoprenes D. The formation of farnesyl pyrophosphate E. Condensation of six activated isoprene units to form squalene

The answer is B. The reduction of HMG-CoA to mevalonate is catalyzed by HMG-CoA reductase, which is an integral enzyme in the smooth endoplasmic reticulum. The activity of HMG-CoA reductase is carefully controlled and is the rate-limiting step for de novo cholesterol biosynthesis. The other steps listed, although they are on the pathway to cholesterol, are not catalyzed by regulated enzymes.

A patient diagnosed with thiamine deficiency exhibited fatigue and muscle cramps. The muscle cramps have been related to an accumulation of metabolic acids. Which of the following metabolic acids is most likely to accumulate in a thiamine deficiency? A. Isocitric acid B. Pyruvic acid C. Succinic acid D. Malic acid E. Oxaloacetic acid

The answer is B. Thiamine pyrophosphate is the coenzyme for the alpha-ketoglutarate dehydrogenase and pyruvate dehydrogenase complexes. With these complexes inactive, pyruvic acid and alpha-ketoglutaric acid accumulate and dissociate to generate the anion and H+. As alpha-ketoglutarate is not listed as an answer, the only possible answer is pyruvate.

The ATP yield from the complete oxidation of 1 mol of a C18:0 fatty acid to carbon dioxide and water would be closest to which one of the following? A. 105 B. 115 C. 120 D. 125 E. 130

The answer is C. An 18-carbon saturated fatty acid would require eight spirals of fatty acid oxidation, which yields 8 NADH, 8 FAD(2H), and 9 acyl-CoA. As each NADH gives rise to 2.5 ATP, and each FAD(2H) gives rise to 1.5 ATP, the reduced cofactors will give rise to 32 ATP. Each acyl-CoA gives rise to 10 ATP, for a total of 90 ATP. This then yields 122 ATP, but we must subtract 2 ATP for the activation step, at which two highenergy bonds are broken. Thus, the net yield is 120 ATPs for each molecule of fatty acid oxidized.

Which of the following is a characteristic of sphingosine? A. It is converted to ceramide by reacting with a UDPsugar. B. It contains a glycerol moiety. C. It is synthesized from palmitoyl-CoA and serine. D. It is a precursor of cardiolipin. E. It is only synthesized in neuronal cells.

The answer is C. Sphingosine is derived from the condensation of palmitoyl-CoA and serine. It is converted to ceramide via reaction with a fatty acyl-CoA, not a UDP-sugar. There is no glycerol in this structure, and cardiolipin is derived from phosphatidic acid and phosphatidylglycerol. All cells synthesize sphingosine; its synthesis is not restricted to neuronal cells.

An adolescent patient with a deficiency of muscle phosphorylase was examined while exercising his or her forearm by squeezing a rubber ball. Compared with a normal person performing the same exercise, this patient would exhibit which of the following? A. Exercise for a longer time without fatigue. B. Have increased glucose levels in blood drawn from his or her forearm. C. Have decreased lactate levels in blood drawn from his or her forearm. D. Have lower levels of glycogen in biopsy specimens from his or her forearm muscle. E. Hyperglycemia

The answer is C. The patient has McArdle disease, a glycogen storage disease caused by a deficiency of muscle glycogen phosphorylase. Because he or she cannot degrade glycogen to produce energy for muscle contraction, he or she becomes fatigued more readily than a normal person (thus, A is incorrect), the glycogen levels in her muscle will be higher than normal as a result of the inability to degrade them (thus, D is incorrect), and his or her blood lactate levels will be lower because of the lack of glucose for entry into glycolysis. He or she will, however, draw on the glucose in his or her circulation for energy, so his or her forearm blood glucose levels will be decreased (thus, B is incorrect), and because the liver is not affected, blood glucose levels can be maintained by liver glycogenolysis (thus, E is incorrect).

Of the major risk factors for the development of atherosclerotic cardiovascular disease (ASCVD), such as sedentary lifestyle, obesity, cigarette smoking, diabetes mellitus, hypertension, and hyperlipidemia, which one, if present, is the only risk factor in a given patient without a history of having had a myocardial infarction that requires that the therapeutic goal for the serum LDL cholesterol level be <100 mg/dL? A. Obesity B. Cigarette smoking C. Diabetes mellitus D. Hypertension E. Sedentary lifestyle

The answer is C. The presence of chronic hyperglycemia (usually accompanied by high levels of free fatty acids in the blood) causes diffuse multiorgan toxic effects ("glucose toxicity" and "lipotoxicity"), to the extent that it raises the risk for a future atherosclerotic event to a level equal to that posed by a history of the patient having already suffered such an event in the past. The other risk factors, in the absence of a previous myocardial infarction, do not require the suggested lower limits for circulating cholesterol levels.

Considering the final steps in cholesterol biosynthesis, when squalene is eventually converted to lanosterol, which of the following statements is correct? A. All of the sterols have three fused rings (the steroid nucleus) and are alcohols with a hydroxyl group at C3. B. The action of squalene monooxygenase oxidizes C14 of the squalene chain, forming an epoxide. C. Squalene monooxygenase is considered a mixedfunction oxidase because it catalyzes a reaction in which only one of the oxygen atoms of O2 is incorporated into the organic substrate. D. Squalene monooxygenase uses reduced flavin nucleotides, such as FAD(2H), as the cosubstrate in the reaction. E. Squalene is joined at carbons 1 and 30 to form the fused-ring structure of sterols.

The answer is C. The squalene monooxygenase reaction uses one of the oxygen atoms of O2 to form an epoxide at one end of the squalene molecule, which requires NADPH as a cosubstrate of the reaction. All sterols have four fused rings (thus, A is incorrect). Because the epoxide is formed at one end of the squalene molecule, answer B is incorrect. Answer D is incorrect because NADPH provides the reducing power, not FAD(2H); and answer E is incorrect because carbons 1 and 30 remain independent during the cyclization of squalene.

An alcoholic is brought to the emergency room in a hypoglycemic coma. Because alcoholics are frequently malnourished, which of the following enzymes can be used to test for a thiamine deficiency? A. Aldolase B. Transaldolase C. Transketolase D. Glucose-6-phosphate dehydrogenase E. UDP-galactose epimerase

The answer is C. Transketolase requires thiamine pyrophosphate as a cofactor, whereas none of the other enzymes listed does. Thus, if an individual has a thiamine deficiency, transketolase activity as isolated from a patient's blood cells will be enhanced by the addition of thiamine; in well-nourished individuals, the addition of thiamine will not enhance transketolase activity.

Your 27-year-old male patient, with a BMI of 34, has a total cholesterol of 450 mg/dL and triglycerides of 610 mg /dL. He exhibits planar xanthomas and has already had one angioplasty last year. This patient may be exhibiting a rare autosomal recessive disorder which generates a mutation in which of the following proteins? (A) LPL (B) Apolipoprotein CII (C) Apolipoprotein E (D) Apolipoprotein B100 (E) Apolipoprotein B48

The answer is C: Apolipoprotein E. The patient has dysbetalipoproteinemia, a mutation in apolipoprotein E, such that the patient exhibits the rare E2 form instead of the normal E3 form. Apolipoprotein E has affinity for the LDL receptor and the LDL receptor-related protein and, as such, is important for chylomicron remnant and IDL uptake from the circulation by the liver. With the homozygous E2 form, binding of the particles to their receptors is weak, and the particles circulate longer than normal, contributing to the high cholesterol and triglyceride levels seen in the circulation. Only about 10% of the individuals who are homozygous for E2 will develop this condition, and in those, obesity (BMI of 34) is a key factor which links the condition to the mutation. This disorder is not a problem with lipoprotein lipase (LPL) digesting triglycerides from particles, so neither LPL nor apo CII is defective. As both chylomicrons and VLDL are produced, it is not a defect in either apo B48 or B100 production or function.

Coenzyme A is synthesized from which of the following vitamins? A. Niacin B. Riboflavin C. Vitamin A D. Pantothenate E. Vitamin C

The answer is D. Pantothenate is the vitamin precursor of coenzyme A. Niacin is the vitamin precursor of NAD, and riboflavin is the vitamin precursor of FAD and FMN. Vitamins A and C are used with only minor modifications, if any.

An individual with a deficiency of an enzyme in the pathway for carnitine synthesis is not eating adequate amounts of carnitine in the diet. Which of the following effects would you expect during fasting as compared with an individual with an adequate intake and synthesis of carnitine? A. Fatty acid oxidation is increased. B. Ketone body synthesis is increased. C. Blood glucose levels are increased. D. Levels of dicarboxylic acids in the blood are increased. E. Levels of very long-chain fatty acids in the blood are increased.

The answer is D. A lack of carnitine would lead to an inability to transport fatty acyl-CoAs into the mitochondria. This would lead to a decrease in fatty acid oxidation (thus, A is incorrect), a decrease in ketone body production because fatty acids cannot be oxidized (thus, B is incorrect), a decrease in blood glucose levels because gluconeogenesis is impaired as a result of a lack of energy (thus, C is incorrect), and no increase in the levels of very long-chain fatty acids because these are initially oxidized in the peroxisomes and do not require carnitine for entry into that organelle (thus, E is incorrect). The beta-oxidation system, which creates dicarboxylic acids, is found in the endoplasmic reticulum, and as the concentration of fatty acyl-CoAs increase in tissues, they will be oxidized by this alternative pathway.

Starting with glyceraldehyde 3-phosphate and synthesizing one molecule of pyruvate, the net yield of ATP and NADH would be which of the following? A. 1 ATP, 1 NADH B. 1 ATP, 2 NADH C. 1 ATP, 4 NADH D. 2 ATP, 1 NADH E. 2 ATP, 2 NADH F. 2 ATP, 4 NADH G. 3 ATP, 1 NADH H. 3 ATP, 2 NADH I. 3 ATP, 4 NADH

The answer is D. By starting with glyceraldehyde 3-phosphate, the energy-requiring steps of glycolysis are bypassed. Thus, as glyceraldehyde 3-phosphate is converted to pyruvate, two molecules of ATP will be produced (at the phosphoglycerate kinase and pyruvate kinase steps) and one molecule of NADH will be produced (at the glyceraldehyde-3-phosphate dehydrogenase step)

A woman was told by her physician to go on a low-fat diet. She decided to continue to consume the same number of calories by increasing her carbohydrate intake while decreasing her fat intake. Which of the following blood lipoprotein levels would be decreased as a consequence of her diet? A. VLDL B. IDL C. LDL D. Chylomicrons E. HDL

The answer is D. Chylomicrons are blood lipoproteins produced from dietary fat. VLDL is produced mainly from dietary carbohydrate. IDL and LDL are produced from VLDL. HDL does not transport triacylglycerol to the tissues.

Dinitrophenol acts as an uncoupler of oxidative phosphorylation by which of the following mechanisms? A. Activating the H+-ATPase B. Activating coenzyme Q C. Blocking proton transport across the inner mitochondrial membrane D. Allowing for proton exchange across the inner mitochondrial membrane E. Enhancing oxygen transport across the inner mitochondrial membrane

The answer is D. Dinitrophenol equilibrates the proton concentration across the inner mitochondrial membrane, thereby destroying the proton motive force. Thus, none of the other answers is correct.

How many moles of ATP are generated by the complete aerobic oxidation of 1 mol of glucose to 6 mol of CO2? A. 2 to 4 B. 10 to 12 C. 18 to 22 D. 30 to 32 E. 60 to 64

The answer is D. Each mole of glucose produces 2 mol of pyruvate, 2 mol of ATP, and 2 mol of NADH during glycolysis. As each pyruvate is converted to acetyl-CoA, 1 NADH is produced. Each acetyl-CoA enters the TCA cycle to produce 1 GTP, 3 NADH, and 1 FAD(2H). Therefore, the oxidation of 1 mol of glucose would yield a total of 2 mol of ATP and 2 mol of NADH from glycolytic reactions; 2 mol of NADH from the conversion of 2 mol of pyruvate to 2 mol of acetyl-CoA; and 6 mol of NADH, 2 mol of GTP, and 2 mol of FAD(2H) from the oxidation of 2 mol of acetylCoA in the TCA cycle. Each GTP has the same energy as 1 ATP. Each mitochondrial NADH is oxidized to produce 2.5 ATP. Each cytoplasmic NADH can generate either 1.5 ATP (using the glycerol phosphate shuttle) or 2.5 ATP (using the malate-aspartate shuttle). Each mitochondrial FAD(2H) is oxidized to produce 1.5 ATP. Therefore, the total ATP yield from the oxidation of 1 mol of glucose is 30 to 32 mol of ATP (30 mol if the glycerol phosphate shuttle is used; 32 mol if the malate-aspartate shuttle is used).

The conversion of nascent chylomicrons to mature chylomicrons requires which of the following? A. Bile salts B. 2-Monoacylglycerol C. Lipoprotein lipase D. High-density lipoprotein E. Lymphatic system

The answer is D. High-density lipoproteins transfer apoproteins CII and E to nascent chylomicrons to convert them to mature chylomicrons. Bile salts are required to emulsify dietary lipid, 2-monoacylglycerol is a digestion product of pancreatic lipase, lipoprotein lipase digests triglyceride from mature chylomicrons, and the lymphatic system delivers the nascent chylomicrons to the bloodstream.

A patient has large deposits of liver glycogen, which after an overnight fast, had shorter-than-normal branches. This abnormality could be caused by a defective form of which one of the following proteins or activities? A. Glycogen phosphorylase B. Glucagon receptor C. Glycogenin D. Amylo-1,6-glucosidase E. Amylo-4,6-transferase

The answer is D. If, after fasting, the branches were shorter than normal, glycogen phosphorylase must be functional and capable of being activated by glucagon (thus, A and B are incorrect). The branching enzyme (amylo-4,6-transferase) is also normal because branch points are present within the glycogen (thus, E is incorrect). Because glycogen is also present, glycogenin is present in order to build the carbohydrate chains, indicating that C is incorrect. If the debranching activity is abnormal (the amylo-1,6-glucosidase), glycogen phosphorylase would break the glycogen down up to four residues from branch points and would then stop. With no debranching activity, the resultant glycogen would contain the normal number of branches, but the branched chains would be shorter than normal.

Type III hyperlipidemia is caused by a deficiency of apoprotein E. Analysis of the serum of patients with this disorder would exhibit which of the following? A. An absence of chylomicrons after eating B. Higher than normal levels of VLDL after eating C. Normal triglyceride levels D. Elevated triglyceride levels E. Lower than normal triglyceride levels

The answer is D. Nascent chylomicrons would be synthesized, which can only acquire apo CII from HDL (thus, A is incorrect). The chylomicrons would be degraded in part by lipoprotein lipase, leading to chylomicron remnant formation. However, the chylomicron remnants would remain in circulation because of the lack of apo E (thus, B is incorrect). Because these remnant particles still contain a fair amount of triglyceride, serum triglyceride levels will be elevated (thus, C and E are incorrect).

A lack of the enzyme ETF-QO oxidoreductase leads to death. This is caused by which of the following reasons? A. The energy yield from glucose use is dramatically reduced. B. The energy yield from alcohol use is dramatically reduced. C. The energy yield from ketone body use is dramatically reduced. D. The energy yield from fatty acid use is dramatically reduced. E. The energy yield from glycogen use is dramatically reduced.

The answer is D. The ETF:CoQ oxidoreductase is required to transfer the electrons from the FAD(2H) of the acyl-CoA dehydrogenase to coenzyme Q. When the oxidoreductase is missing, the electrons cannot be transferred, and the acyl-CoA dehydrogenase cannot continue to oxidize fatty acids because it has a reduced cofactor instead of an oxidized cofactor. During times of fasting, when fatty acids are the primary energy source, no energy will be forthcoming, gluconeogenesis is shut down, and death may result. The lack of this enzyme does not affect the other pathways listed as potential answers.

Consider the following experiment. Carefully isolated liver mitochondria are placed in a weakly buffered solution. Malate is added as an energy source, and an increase in oxygen consumption confirms that the electron-transport chain is functioning properly within these organelles. Valinomycin and potassium are then added to the mitochondrial suspension. Valinomycin is a drug that allows potassium ions to freely cross the inner mitochondrial membrane. What is the effect of valinomycin on the proton motive force that had been generated by the oxidation of malate? A. The proton motive force will be reduced to a value of zero. B. There will be no change in the proton motive force. C. The proton motive force will be increased. D. The proton motive force will be decreased but to a value greater than zero. E. The proton motive force will be decreased to a value less than zero.

The answer is D. The proton motive force consists of two components: a delta pH and a delta Psi (electrical component). The addition of valinomycin and potassium will destroy the electrical component but not the pH component. Thus, the proton motive force will decrease but will still be greater than zero. Thus, the other answers are all incorrect.

A molecule of palmitic acid, attached to carbon 1 of the glycerol moiety of a triacylglycerol, is ingested and digested. The fatty acid is stored in a fat cell and ultimately is oxidized to carbon dioxide and water in a muscle cell. Choose the molecular complex in the blood in which the palmitate residue is carried from the lumen of the gut to the surface of the gut epithelial cell. A. VLDL B. Chylomicron C. Fatty acid-albumin complex D. Bile salt micelle E. LDL

The answer is D. The triacylglycerol is degraded by pancreatic lipase, which releases the fatty acids at positions 1 and 3. The fatty acids released are then transported to the cell surface in a bile salt micelle. The only exception is short-chain fatty acids (shorter than palmitic acid), which can diffuse to the cell surface and enter the intestinal cell in the absence of micelle formation.

Patients with von Gierke disease display hepatomegaly. Glycogen content in the liver is increased, relative to normal, due to which of the following effects of glucose6-phosphate in these patients? (A) Inhibition of phosphorylase a (B) Stimulation of phosphorylase b (C) Inhibition of glycogen synthase I (D) Stimulation of glycogen synthase D (E) Inhibition of glycogen phosphorylase kinase

The answer is D: Stimulation of glycogen synthase D. Glycogen synthase D (the inactive, phosphorylated form) can be allosterically activated by glucose-6-phosphate binding to the enzyme. Glucose-6-phosphate will inhibit the AMP-stimulation of muscle phosphorylase b, but does not have any allosteric effect on the other enzymes listed (PFK-1, glucose-6-phosphatase, or GLUT4 transporters) as answer choices for this problem.

In which one of the following periods will fatty acids be the major source of fuel for the tissues of the body? A. Immediately after breakfast B. Minutes after a snack C. Immediately after dinner D. While running the first mile of a marathon E. While running the last mile of a marathon

The answer is E. Fatty acids are the major fuel for the body during prolonged exercise and fasting. Answers A, B, and C are incorrect because glucose would be the major fuel after eating. Answer D is incorrect because, at the start of exercise, muscle glycogen and gluconeogenesis are being used as the major source of fuel.

During exercise, stimulation of the TCA cycle results principally from which of the following? A. Allosteric activation of isocitrate dehydrogenase by increased NADH B. Allosteric activation of fumarase by increased ADP C. A rapid decrease in the concentration of four-carbon intermediates D. Product inhibition of citrate synthase E. Stimulation of the flux through a number of enzymes by a decreased NADH/NAD+ ratio

The answer is E. NADH decreases during exercise to generate energy for the exercise (if it were increased, it would inhibit the cycle and slow it down); thus the NADH/NAD+ ratio is decreased; and the lack of NADH activates flux through isocitrate dehydrogenase, alpha-ketoglutarate dehydrogenase, and malate dehydrogenase. Isocitrate dehydrogenase is inhibited by NADH; so, answer A is not correct. Fumarase is not regulated, thus answer B is incorrect. The four-carbon intermediates of the cycle are regenerated during each turn of the cycle, so their concentrations do not decrease (thus, C is incorrect). Product inhibition of citrate synthase would slow the cycle and not generate more energy; hence, D is incorrect.

Which of the following coenzymes is unique to alpha-keto acid dehydrogenase complexes? A. NAD+ B. FAD C. GDP D. H2O E. Lipoic acid

The answer is E. The alpha-keto acid dehydrogenase complex uses thiamine pyrophosphate, lipoic acid, FAD, NAD+, and CoASH. Of these, only lipoate is used by no other enzyme and is unique to the alpha-keto acid dehydrogenase complexes. Many dehydrogenases use NAD+ as a coenzyme (such as malate dehydrogenase) or FAD (such as succinate dehydrogenase) (thus, A and B are incorrect). Answer C is incorrect because GDP is not a coenzyme for alpha-keto acid dehydrogenase complexes. Answer D is incorrect because H2O is not a coenzyme.

Asians and Native Americans may flush and feel ill after drinking small amounts of ethanol in alcoholic beverages. This reaction is due to genetic variation in an enzyme that metabolizes the liver metabolite of alcohol, which is which of the following? a. Methanol b. Acetone c. Acetaldehyde d. Hydrogen peroxide e. Glycerol

The answer is c. (Murray, pp 212-218. Scriver, pp 1521-1552.) The principal pathway for hepatic metabolism of ethanol is thought to be oxidation to acetaldehyde in the cytoplasm by alcohol dehydrogenase. Acetaldehyde is then oxidized, chiefly by acetaldehyde dehydrogenase within the mitochondrion, to yield acetate. Acetone, methanol, hydrogen peroxide, and glycerol do not appear in this biodegradation pathway. The genetic variations of acetaldehyde dehydrogenase have few phenotypic effects aside from sensitivity to alcoholic beverages and are extremely common in the affected populations. These characteristics qualify acetaldehyde dehydrogenase variation as an example of enzyme polymorphism.

Newly synthesized fatty acids are not immediately degraded because of which of the following? A. Tissues that synthesize fatty acids do not contain the enzymes that degrade fatty acids. B. High NADPH levels inhibit Beta-oxidation. C. In the presence of insulin, the key fatty acid degrading enzyme is not induced. D. Newly synthesized fatty acids cannot be converted to their CoA derivatives. E. Transport of fatty acids into mitochondria is inhibited under conditions in which fatty acids are being synthesized.

The answer is E. When fatty acids are being synthesized, malonyl-CoA accumulates, which inhibits carnitine palmitoyl transferase I. This blocks fatty acid entry into the mitochondrion for oxidation. Many tissues both synthesize and degrade fatty acids (such as liver and muscle; thus, A is incorrect). NADPH blocks the glucose6-phosphate dehydrogenase reaction but not fatty acid oxidation (thus, B is incorrect). Insulin has no effect on the synthesis of the enzymes involved in fatty acid degradation (unlike the effect of insulin on the induction of enzymes involved in fatty acid synthesis; thus, C is incorrect). Finally, newly synthesized fatty acids are converted to their CoA derivatives for elongation and desaturation (thus, D is incorrect).

When glycogen is degraded, glucose 1-phosphate is formed. Glucose 1-phosphate can then be isomerized to glucose 6-phosphate. Starting with glucose 1-phosphate and ending with two molecules of pyruvate, what is the net yield of glycolysis in terms of ATP and NADH formed? A. 1 ATP, 1 NADH B. 1 ATP, 2 NADH C. 1 ATP, 3 NADH D. 2 ATP, 1 NADH E. 2 ATP, 2 NADH F. 2 ATP, 3 NADH G. 3 ATP, 1 NADH H. 3 ATP, 2 NADH I. 3 ATP, 3 NADH

The answer is H. Glucose 1-phosphate is isomerized to glucose 6-phosphate, which then enters glycolysis. This skips the hexokinase step, which uses 1 ATP. Thus, starting from glucose 1-phosphate, one would get the normal 2 ATP and 2 NADH, but with one less ATP used, for a total yield of 3 ATP and 2 NADH.

Certain carbohydrates can be recognized as reducing substances in urine by the Clinitest reaction, a tablet that combines with sugars to form a green color reaction. Among these is glucose with a C1 aldehyde group and four asymmetric carbons that generate 16 isomeric forms including galactose and mannose. Other hexose isomers have a ketone group at C2 (ketoses), which like the aldehyde can produce a reduction reaction. Which of the following is a ketose isomer of glucose? a. Fructose b. Galactose c. Glucofuranose d. Glucopyranose e. Mannose

The answer is a. (Murray, pp 102-110.) Glucofuranose and glucopyranose are ring structures of glucose, with the majority of glucose in solution in the glucopyranose form. Galactose and mannose are epimers of glucose (an aldose), and fructose is the ketose isomer of glucose.

Which of the following is an example of a ketose sugar? a. Fructose b. Galactose c. Glucose d. Ribose e. Xylose

The answer is a. (Murray, pp 102-110.) Glucose can form 16 different isomers. The orientation of the H and OH groups on the carbon atom adjacent to the CH2OH group determines whether the sugar is the D- or L-isomer. Most of the monosaccharides in mammals are in the D form. Glucose can also form a ring structure with either a six-membered ring (glucopyranose) or a five-membered ring (glucofuranose). Epimers of glucose are isomers that differ in their configuration of the H and OH groups at the 2, 3, and 4 carbons. Mannose and galactose are the most biologically important epimers of glucose. Fructose is an isomer of glucose in which there is a keto group at the anomeric 2 carbon. Glucose is an aldose sugar with an aldehyde group at the anomeric carbon in position 1.

Which of the following is an energy-requiring step of glycolysis? a. Glucokinase b. Lactate dehydrogenase c. Phosphoglycerate kinase d. Pyruvate kinase

The answer is a. (Murray, pp 136-144. Scriver, pp 1433-1436.) Glucokinase catalyzes the conversion of glucose to glucose-6-phosphate in the energy-requiring first step of glycolysis. ATP is also required in the conversion of fructose-6-phosphate to fructose 1,6-bisphosphate by phosphofructokinase. ATP is generated in the conversion of 1,3-bisphosphoglycerate to 3-phosphoglycerate by phosphoglycerate kinase and in the conversion of phophoenolpyruvate to pyruvate by pyruvate kinase.

A 6-month-old male becomes ill after fruits and vegetables are added to his diet of breast milk. Mother feels that he used to become colicky when she ate fruit, although her pediatrician did not think this was significant. After 1 month of these new foods, the child has stopped gaining weight and the pediatrician feels an enlarged liver. Initial blood tests show a mild acidosis (pH 7.2) with increased lactic acid and low blood glucose. The Clinitest reaction is positive for reducing substances in the urine, but the glucose oxidase test is negative for glucosuria. A glycogen storage disease is suspected, and a liver biopsy dose shows mildly increased glycogen with marked cellular damage suggestive of early cirrhosis. Assays for type IV glycogen storage disease are negative (Table 3), and the initial frozen urine sample is reanalyzed and found to contain fructose. The most likely diagnosis and the reasons for hypoglycemia and glycogen accumulation is which of the following? a. Hereditary fructose intolerance with inhibition of liver phosphorylase b. Hereditary fructose intolerance with inhibition of glycogen synthase c. Essential fructosuria with inhibition of glycogen synthase d. Essential pentosuria with inhibition of liver phosphorylase e. Essential fructosuria with allosteric stimulation of glycogen synthase

The answer is a. (Murray, pp 136-144. Scriver, pp 1489-1520.) Hereditary fructose intolerance (229600) is caused by deficiency of aldolase B that converts fructose-1-phosphate to dihydroxyacetone phosphate and glyceraldehydes. Fructose can be converted to fructose-1-phosphate (by fructokinase, the block in essential fructosuria, 229800) but accumulates with its phosphate and is diverted to fructose-1,6-bisphosphate. These compounds allosterically inhibit glycogen phosphorylase and cause hypoglycemia in the presence of abundant glycogen stores. The abnormal sequestration of phosphate interferes with ATP generation from AMP, depleting cellular energy sources with severe effects on liver or kidney. Affected individuals become nauseated when eating fructose and exhibit a natural aversion to fruits. If diagnosis is postponed and fructose is not minimized in the diet, they can undergo progressive liver and kidney failure with malnutrition and death. Countries like Belgium that use fructose in hyperalimentation solutions may observe patients with milder fructose intolerance who decompensate in the face of high serum concentrations.

An infant with hypoglycemia and a palpable liver is evaluated for possible glycogen storage disease. The parents have immigrated from Russia, and report that the child's older brother was diagnosed with a "debrancher" enzyme deficiency with similar glycogen storage. This diagnosis would imply accumulation of glycogen with which type of glucose linkages? a. Linear α1→4 linkages with branching α1→6 linkages b. Linear α1→6 linkages with branching β1→4 linkages c. Linear β1→4 linkages only d. Linear β1→6 linkages only e. Branching β1→6 linkages only

The answer is a. (Murray, pp 145-152. Scriver, pp 1521-1552.) Normal glycogen is composed of glucose residues joined in straight chains byα1→4 linkages. At 4- to 10-residue intervals, a branch of α1→4 linkages is initiated at an α1 → 6 linkage. Glycogen particles can contain up to 60,000 glucose residues. In the absence of the debrancher enzyme, glycogen can be degraded only to the branch points, inhibiting release of glucose into the serum and causing glycogen storage. As noted in HighYield Facts Table 4, Forbes/Cori or type 3 glycogen storage disease (232400) involves deficiency of debranching enzyme.

Which enzyme reaction of the citric acid cycle leads to production of ATP (or GTP) by substrate-level phosphorylation? a. Aconitase b. Citrate synthase c. Fumarase d. Isocitrate dehydrogenase e. α-ketoglutarate dehydrogenase f. Malate dehydrogenase g. Succinate dehydrogenase h. Succinate thiokinase

The answer is h. (Murray, pp 130-135. Scriver, pp 2327-2356.) The citric acid cycle produces 12 ATP per turn. Most of these ATP are generated by reoxidation of the NADH and FADH2 molecules produced by the dehydrogenases. Isocitrate dehydrogenase, α-ketoglutarate dehydrogenase, and malate dehydrogenase each produce one NADH. Succinate dehydrogenase produces one molecule of FADH2 per turn of the cycle. Reoxidation of each NADH results in formation of 3 ATP, and reoxidation of FADH2 results in production of 2 ATP. Succinate thiokinase is the only enzyme that generates ATP directly by substrate-level phosphorylation.

A frequent presentation in the newborn period is transient hypoglycemia as the child adapts to separation from maternal glucose controls. Blood glucose is generally maintained at concentrations of 4.5-5.5 mmol/L but may rise to 6.5-7.2 mmol/L after feeding or decrease to 3.3-3.9 mmol/L in the fasting state. Which of the following enzymes plays an important role in regulating blood glucose levels after feeding? a. Glucokinase b. Glucose-6-phosphatase c. Phosphofructokinase d. Pyruvate kinase

The answer is a. (Murray, pp 153-162. Scriver, pp 14171-1488.) Glucokinase promotes uptake of large amounts of glucose by the liver. At normal glucose levels, the liver produces glucose from glycogen, but as glucose levels rise after feeding, the liver stops converting glycogen and instead takes up glucose. Insulin also plays a role in regulating blood glucose levels. Pancreatic β-cells produce insulin in response to hyperglycemia. Glucose uptake by the β-cells and phosphorylation by glucokinase stimulates secretion of insulin, which enhances glucose transport into adipose tissues and muscle and thus lowers blood glucose levels.

A Nigerian medical student studying in the United States develops hemolytic anemia after taking the oxidizing antimalarial drug primaquine. Which of the following is the most likely cause of this severe reaction? a. Glucose-6-phosphate dehydrogenase deficiency b. Concomitant scurvy c. Vitamin C deficiency d. Diabetes e. Glycogen phosphorylase deficiency

The answer is a. (Murray, pp 163-170, 613-619. Scriver, pp 4517-4554.) One of the world's most common enzyme deficiencies is glucose-6-phosphate-dehydrogenase deficiency (305900). This deficiency in erythrocytes is particularly prevalent among African and Mediterranean males. A deficiency in glucose-6-phosphate dehydrogenase blocks the pentose phosphate pathway and NADPH production. Without NADPH to maintain glutathione in its reduced form, erythrocytes have no protection from oxidizing agents. This X-linked recessive deficiency is often diagnosed when patients develop hemolytic anemia after receiving oxidizing drugs such as primaquine or after eating oxidizing substances such as fava beans.

Diarrhea from infection or malnutrition is the world's most prevalent killer of children. A child develops chronic diarrhea and liver inflammation in early infancy when the mother begins using formula that includes corn syrup. Evaluation of the child demonstrates sensitivity to fructose in the diet. Which of the following glycosides contains fructose and therefore should be avoided when feeding or treating this infant? a. Sucrose b. Oaubain c. Lactose d. Maltose e. Streptomycin

The answer is a. (Murray, pp 163-172. Scriver, pp 1489-1520.) Glycosides are formed by condensation of the aldehyde or ketone group of a carbohydrate with a hydroxyl group of another compound. Other linked groups (aglycones) include steroids with hydroxyl groups (e.g., cardiac glycosides such as digitalis or ouabain) or other chemicals (e.g., antibiotics such as streptomycin). Sucrose (α-D-glucose-β-1 → 2-D-fructose), maltose (α-D-glucose-α-1 → 4-D-glucose), and lactose (α-D-galactose-β-1 → 4-Dglucose) are important disaccharides. Fructose is among several carbohydrate groups known as ketoses because it possesses a ketone group. The ketone group is at carbon 2 in fructose, and its alcohol group at carbon 1 (also at carbon 6) allows ketal formation to produce pyranose and furanose rings as with glucose. Most of the fructose found in the diet of North Americans is derived from the disaccharide sucrose (common table sugar). Sucrose is cleaved into equimolar amounts of glucose and fructose in the small intestine by the action of the pancreatic enzyme sucrase. Deficiency of sucrase can also cause chronic diarrhea. Hereditary fructose intolerance (229600) is caused by deficiency of the liver enzyme aldolase B, which hydrolyzes fructose-1-phosphate.

A normal female infant begins having jittery spells, vomiting, and falloff in growth when introduced to fruits and vegetables at age 6 months. Serum tests reveal low glucose and increased blood lactate, and her physician suspects hereditary fructose intolerance (229600), which is a deficiency of the enzyme aldolase B. The symptoms and serum abnormalities of this disease are due to which of the following? a. Accumulation of hexose phosphates, phosphate and ATP depletion, defective electron transport, and glycogen phosphorylase inhibition b. Accumulation of triose phosphates, phosphate and ATP excess, defective glycolysis, and glycogen synthase inhibition c. Accumulation of triose phosphates, phosphate and ATP depletion, defective electron transport, and glycogen synthase inhibition d. Accumulation of hexose phosphates, phosphate and ATP depletion, defective electron transport, and glycogen phosphorylase stimulation e. Accumulation of hexose phosphates, phosphate and ATP excess, defective electron transport, and glycogen phosphorylase stimulation

The answer is a. (Murray, pp 163-172. Scriver, pp 1489-1520.) Hereditary fructose intolerance (229600) is a defect in aldolase B, causing accumulation of fructose-1-phosphate and other hexose phosphates. The accumulated hexose phosphates deplete cellular phosphate pools, inhibiting generation of ATP through glycolysis or oxidative-phosphorylation (with increased lactate). Altered AMP/ATP ratios cause increased uric acid formation, and inhibition of glycogen phosphorylase by fructose phosphates produces hypoglycemia. Once recognized, hereditary fructose intolerance can be treated by elimination of fructose and sucrose from the diet.

Which of the following explains why individuals with hyperlipidemia and/or gout should minimize their intake of sucrose and highfructose syrups? a. Fructose is initially phosphorylated by liver fructokinase b. After initial modification, fructose is cleaved by a specific enolase c. Fructose is converted to UDP-fructose d. Fructose is ultimately converted to galactose e. Fructose can be phosphorylated by hexokinase in adipose cells

The answer is a. (Murray, pp 163-172. Scriver, pp 1521-1552.) Fructose is taken in by humans as sucrose, sucrose-containing syrups, and the free sugar. Fructose is mainly phosphorylated to fructose-1-phosphate by liver fructokinase. Aldol cleavage by fructose-1-phosphate-specific aldolase, not enolase, yields glyceraldehyde and dihydroxyacetone phosphate. The glyceraldehyde is phosphorylated to glyceraldehyde-3-phosphate by triose kinase, and both triose phosphates can enter glycolysis. Excess fructose from commercial foods can exercise adverse effects by raising blood lipids and uric acid. Fructose phosphorylation bypasses phosphofructokinase, a regulatory enzyme of glycolysis and provides excess glycerol metabolites and excess triglyceride/lipid biosynthesis. Fructose phosphorylation can also deplete liver cell ATP, lessening its inhibition of adenine nucleotide degradation and increasing production of uric acid. In adipocytes, fructose can be alternatively phosphorylated by hexokinase to fructose-6-phosphate. However, this reaction is competitively inhibited by appreciable amounts of glucose, as it is in other tissues.

A child has ingested cyanide from her parents' garage and is rushed to the emergency room. Which of the following components of the citric acid cycle will be depleted first in this child? a. NAD+ cofactor b. Citrate synthase c. Aconitase d. Citrate production e. Acetyl-CoA production

The answer is a. (Murray, pp 92-101, 130-135. Scriver, pp 2261-2274.) Cyanide blocks respiration by displacing oxygen from hemoglobin. Oxidative phosphorylation in the mitochondria cannot proceed because cyanide cannot oxidize (remove electrons) from reduced cofactors like NADH. The citric acid cycle is the major pathway for generating ATP and reducing equivalents (NADH, H+) from catabolism of carbohydrates, amino acids, and lipids. Inability to regenerate NAD+ from NADH through mitochondrial oxidative phosphorylation depletes the cell of NAD+ and inhibits the citric acid cycle. Failure to generate ATP by oxidative phosphorylation using NADH from the citric acid cycle depletes the cell of energy and leads to cell and tissue death (organ failure). Enzymes (citrate synthase, aconitase) and intermediates of the citric acid cycle (citrate, acetyl coenzyme A) need only be present in trace amounts because they are not consumed.

Which of the following events occurs during formation of phosphoenolpyruvate from pyruvate during gluconeogenesis? a. CO2 is consumed b. Inorganic phosphate is consumed c. Acetyl-CoA is utilized d. ATP is generated e. GTP is generated

The answer is a.(Murray, pp 153-162. Scriver, pp 1521-1552.) In the formation of phosphoenolpyruvate during gluconeogenesis, oxaloacetate is an intermediate. In the first step, catalyzed by pyruvate carboxylase, pyruvate is carboxylated with the utilization of one high-energy ATP phosphate bond: pyruvate + ATP + CO2 → oxaloacetate + ADP + Pi In the second step, catalyzed by phosphoenolpyruvate carboxykinase, a highenergy phosphate bond of GTP drives the decarboxylation of oxaloacetate: oxaloacetate + GTP → phosphoenolpyruvate + GDP + CO2 In contrast to gluconeogenesis, the formation of pyruvate from phosphoenolpyruvate during glycolysis requires only pyruvate kinase, and ATP is produced.

Among the many molecules of high-energy phosphate compounds formed as a result of the functioning of the citric acid cycle, one molecule is synthesized at the substrate level. In which of the following reactions does this occur? a. Citrate →α-ketoglutarate b. α-Ketoglutarate → succinate c. Succinate → fumarate d. Fumarate → malate e. Malate → oxaloacetate

The answer is b. (Murray, pp 130-135. Scriver, pp 1521-1552.) A molecule of guanosine triphosphate is synthesized from guanosine diphosphate and phosphate at the cost of hydrolyzing succinyl-CoA to succinate and CoA. This constitutes substrate-level phosphorylation, and, in contrast to oxidative phosphorylation, this is the only reaction in the citric acid cycle that directly yields a high-energy phosphate bond. The sequence of reactions from α-ketoglutarate to succinate is catalyzed by the α-ketoglutarate dehydrogenase complex and succinyl-CoA synthetase, respectively. α-ketoglutarate + NAD+ + acetyl-CoA → succinyl-CoA + CO2 + NADH succinyl-CoA + Pi + GDP → succinate + GTP + acetyl-CoA

Recent statistics indicate alcohol abuse costs the United States $184 billion annually, more than cancer ($107 billion) or obesity ($100 billion). In 2001, 47% of those between ages 12 and 20 years admitted to drinking, 30% of these to binge drinking in the past month. Chronic alcoholics require more ethanol than do nondrinkers to become intoxicated because of a higher level of a specific enzyme. However, independent of specific enzyme levels, the availability of what other substance is rate-limiting in the clearance of ethanol? a. NADH b. NAD+ c. FADH d. FAD+ e. NADPH

The answer is b. (Murray, pp 136-144. Scriver, pp 1521-1552.) In humans, ethanol is cleared from the body by oxidation catalyzed by two NAD+-linked enzymes: alcohol dehydrogenase and acetaldehyde dehydrogenase. These enzymes act mainly in the liver to convert alcohol to acetaldehyde and acetate, respectively. In chronic alcoholics, alcohol dehydrogenase may be elevated somewhat. The NADH level is significantly increased in the liver during oxidation of alcohol, owing to the consumption of NAD+. This leads to a swamping of the normal means of regenerating NAD+. Thus, NAD+ becomes the rate-limiting factor in oxidation of excess alcohol.

A child is evaluated because of chronic anemia, slightly enlarged spleen, increased reticulocyte count, and mild elevation of indirectreacting (unconjugated) bilirubin in serum. Incubation of the child's red cells with glucose yield decreased amounts of ATP as compared to controls, even in the presence of aeration (added oxygen). The child's anemia is explained by the fact that ATP is produced by which of the following pathways? a. Glycogen breakdown b. Glycolysis c. Oxidative phosphorylation d. Pentose phosphate cycle e. Lactate conversion to glucose (Cori cycle)

The answer is b. (Murray, pp 136-144. Scriver, pp 4637-4664.) Glycolysis is the major source of ATP in cells lacking mitochondria, but is a minor source of ATP in tissues undergoing active oxidative phosphorylation. Anaerobic tissues (like muscle during exercise) or those without mitochondria (like erythrocytes) are dependent on glucose metabolism by glycolysis to lactate (through pyruvate) for production of ATP and energy. Defects in glycolytic enzymes (like hexokinase deficiency—235700) reduce ATP production in erythrocytes, shortening red cell lifespan with increased cell death (hemolysis). The increased hemolysis decreases red cell counts (anemia) with increased heme conversion to bilirubin, increased jaundice, and splenomegaly due to red cell storage. The increased heme load prior to liver metabolism increases indirect-reacting (unconjugated) bilirubin rather than increased conjugated or direct-reacting bilirubin from defective liver/gall bladder metabolism. Lactate produced by red cells and exercising muscle is converted to glucose by the liver, while pyruvate produced during glycolysis can be used for the synthesis of certain amino acids.

What is the role of glucagon? a. To stimulate the citric acid cycle b. To stimulate gluconeogenesis c. To stimulate glycolysis d. To stimulate the pentose phosphate pathway

The answer is b. (Murray, pp 145-152. Scriver, pp 1471-1488.) Glucagon is a hormone in response to decreases in blood glucose. This hormone stimulates gluconeogenesis in the liver by increasing the level of cAMP, which in turn activates the cAMP-dependent protein kinase. This enzyme inactivates pyruvate kinase by phosphorylation and thus inhibits glycolysis.

Children with glycogen storage disease may exhibit symptoms due to altered liver (types I, III, IV, VI, VIII) or muscle (types V, VII) metabolism. Which of the following conversions explains the difference in these presentations? a. Conversion of glycogen to lactate in liver b. Conversion of glycogen and lactate to glucose in liver c. Conversion of glycogen to glucose in muscle d. Conversion of glycogen to alanine in muscle e. Conversion of glycogen to glucose-6-phosphate in liver

The answer is b. (Murray, pp 145-152. Scriver, pp 1521-1551.) Glucose-1-phosphate is the first intermediate in the conversion of glycogen to glucose. The enzyme glycogen phosphorylase catalyzes this first step. The second intermediate, glucose-6-phosphate is subsequently converted to glucose by the enzyme glucose-6-phosphatase. This enzyme is found only in the liver and kidney; thus, these are the only tissues able to break down glycogen for use by other tissues. In tissues such as muscle, glycogen can be broken down to glucose-6-phosphate but can only be used in the cell in which it was produced. Glycolysis in muscle produces lactate, which must be converted to glucose by liver or kidney via the Cori cycle. Defects in liver glycogen metabolism therefore impair glucose-6-phosphate production or gluconeogenesis with resulting hypoglycemia and liver glycogen storage with or without toxicity (cirrhosis). Defects in muscle glycogen metabolism impair contraction (cramps, fatigue) with decreased serum lactate production during exercise and muscle glycogen accumulation (progressive weakness and atrophy).

A child is evaluated for hypoglycemia and lactic acidosis and noted to have an enlarged liver. Biopsy reveals stored glycogen, and a glycogen storage disease is suspected. Assay of usual glycogen enzyme deficiencies in the liver specimen is normal, so the metabolic consultant recommends assay of rarer enzyme deficiencies that influence glycogen metabolism. These enzymes would most likely include which of the following? a. Hexokinase b. cAMP-dependent protein kinase c. Glucose-6-phosphate dehydrogenase d. Phosphofructokinase e. fructose-1,6-diphosphatase

The answer is b. (Murray, pp 145-152. Scriver, pp 1521-1552.) Glycogen storage diseases are a group of inherited enzyme deficiencies that cause accumulation of glycogen in liver, heart, or muscle. Glucose is the primary source of energy for most cells and excess glucose is stored as glycogen. Glycogen provides for short-term high-energy consumption in muscle and is an emergency energy supply for the brain. Glycogen stored in the liver can be converted back to glucose for release into the blood stream for use by other tissues. Glycogen synthesis and breakdown (glycogenolysis) are accomplished by separate pathways rather than reversible reactions. Glycogen synthase is active when dephosphorylated, glycogen phosphorylase when phosphorylated. Cyclic AMP-dependent protein kinases regulate these enzyme phosphorylations, integrating glycogen synthesis/breakdown with food and glucose availability (refer to Fig. 14 in the High-Yield Facts section). Besides deficiencies of phosphorylase (types V, VI) or phosphorylase kinase (VIII), deficiency of adenylyl kinase or cAMP-dependent protein kinase can alter glycogenesis/glycogenolysis regulation and produce glycogen storage (refer to High-Yield Fact Table 4). Deficiency of phosphofructokinase is compensated in liver but can lead to glycogen storage and exercise intolerance in muscle.

After a meal, blood glucose enters cells and is stored as glycogen, particularly in the liver. Which of the following is the donor of new glucose molecules in glycogen? a. UDP-glucose-1-phosphate b. UDP-glucose c. UDP-glucose-6-phosphate d. Glucose-6-phosphate e. Glucose-1-phosphate

The answer is b. (Murray, pp 145-162. Scriver, pp 1521-1552.) Upon entering cells, blood glucose is rapidly converted to glucose-6phosphate by hexokinase or, in the case of the liver, by glucokinase. Glucose6-phosphate is in equilibrium with glucose-1-phosphate via the action of phosphoglucomutase. Glucose-1-phosphate is activated by UTP to form UDP-glucose, which is added to glycogen by an α1→4 linkage in the presence of glycogen synthase. To increase the solubility of glycogen and to increase the number of terminal residues, glycogen-branching enzyme transfers a block of about 7 residues from a chain at least 11 residues long to a branch point at least 4 residues from the last branch point. The branch is attached by an α1→6 linkage.

Which of the following is a primary substrate for gluconeogenesis? a. Galactose b. Glycerol c. Glycogen d. Sucrose

The answer is b. (Murray, pp 153-162. Scriver, pp 1471-1478.) Gluconeogenesis refers to the pathway for converting noncarbohydrate precursors to glucose. Glycerol, lactate, propionate, and certain amino acids such as alanine are all substrates for gluconeogenesis. Glycogen is a glucose storage molecule that can readily be converted in the liver back to glucose for maintenance of blood glucose levels between meals.

Which of the following statements correctly describe human glucose metabolism? a. Liver is impermeable to glucose in the absence of insulin b. Pancreatic β-cells, liver and brain are freely permeable to glucose due to specific glucose transporters c. Liver glucokinase phosphorylates glucose at high rates under all conditions d. Extrahepatic tissues are permeable to glucose when glucagon is present e. Liver takes up glucose when serum glucose is normal but releases it when serum glucose is high

The answer is b. (Murray, pp 157-160; Scriver, pp 1521-1552.) Liver cells are permeable to glucose while extrahepatic tissues require insulin for glucose entry, reflecting different glucose transporters (GLUT) in different tissues. Liver hexokinase has a low Km for glucose and acts at a constant rate, while glucokinase has a higher Km for glucose and promotes glucose uptake at high concentrations as found in the portal vein after meals. The liver releases glucose at normal serum glucose concentrations but takes up glucose at high serum glucose concentrations. Insulin and glucagon act in opposing fashion to regulate serum glucose concentration. Insulin, secreted by the pancreatic β-cell in response to internal increases in glucose, ATP, and calcium influx, increases glucose uptake by muscle and adipose cells by recruiting glucose transporters to their plasma membranes. Glucagon, secreted by the pancreatic α-cells, stimulates cyclic AMP synthesis with increased gluconeogenesis and glycogenolysis to increase serum glucose concentrations.

A newborn begins vomiting after feeding, becomes severely jaundiced, has liver disease, and cloudy lenses of the eyes suggestive of cataracts. Treatment for possible sepsis is initiated, and the urine is found to have reducing substances. A blood screen for galactosemia is positive, and lactose-containing substances are removed from the diet. Lactose is toxic in this case because of which of the following? a. Excess glucose accumulates in the blood b. Galactose is converted to the toxic substance galactitol (dulcitol) c. Galactose competes for glucose during hepatic glycogen synthesis d. Galactose is itself toxic in even small amounts e. Glucose metabolism is shut down by excess galactose

The answer is b. (Murray, pp 163-172. Scriver, pp 1553-1588.) Lactose in breast milk and infant formula is converted by intestinal lactase to glucose and galactose that are efficiently absorbed. In galactosemia (230400), deficiency of galactose-1-phosphate uridyl transferase prevents the conversion of galactose into glucose-6-phosphate by the liver or erythrocytes. Most other organs do not metabolize galactose. The severe symptoms of galactosemia are caused by the reduction of galactose to galactitol (dulcitol) in the presence of the enzyme aldose reductase. High levels of galactitol cause cataracts, the accumulation of galactose-1phosphate contributes to liver disease, and the accumulation of galactose metabolites in urine can be measured as reducing substances by the Clinitest method. Any carbohydrate, including glucose, with a C1 aldehyde registers as a reducing substance by Clinitest, so a Dextrostix (glucose only) test is often performed as a control. In normal children, galactose is first phosphorylated by ATP to produce galactose-1-phosphate in the presence of galactokinase. Next, galactose-1-phosphate uridyl transferase transfers UDP from UDP-glucose to form UDP-galactose and glucose-1-phosphate. Under the action of UDP-galactose-4-epimerase, UDP-galactose is epimerized to UDP-glucose. Finally, glucose-1-phosphate is isomerized to glucose6-phosphate by phosphoglucomutase. Infants with suspected galactosemia must be withdrawn from breast-feeding or lactose formulas and placed on nonlactose formulas such as Isomil.

Following a fad diet meal of skim milk and yogurt, an adult female patient experiences abdominal distention, nausea, cramping, and pain followed by a watery diarrhea. This set of symptoms is observed each time the meal is consumed. Which of the following is the most likely diagnosis? a. Steatorrhea b. Lactase deficiency c. Maltose deficiency d. Sialidase deficiency e. Lipoprotein lipase deficiency

The answer is b. (Murray, pp 474-480. Scriver, pp 1521-1552.) In many populations, a majority of adults are deficient in lactase and hence intolerant to the lactose in milk. In all populations, at least some adults have lactase deficiency (223000). Since virtually all children are able to digest lactose, this deficiency obviously develops in adulthood. In lactase-deficient adults, lactose accumulates in the small intestine because no transports exist for the disaccharide. An outflow of water into the gut owing to the osmotic effect of the milk sugar causes the clinical symptoms. Steatorrhea, or fatty stools, is caused by unabsorbed fat, which can occur following a fatty meal in persons with a deficiency of lipoprotein lipase (238600). Sialidase deficiency (256550) causes accumulation of sialic acid-containing proteoglycans and neurodegeneration.

How many ATP molecules are generated by glycolysis of one glucose molecule? a. One b. Two c. Four d. Six e. Twelve

The answer is b.(Murray, pp 136-144. Scriver, pp 1471-1488.)In the first steps of glycolysis, two ATPs are hydrolyzed, one in the phosphorylation of glucose to glucose-6-phosphate by glucokinase (hexokinase), and one in phosphorylation of fructose-6-phosphate to fructose 1,6-bisphosphate by phosphofructokinase. ATP is generated during conversion of 1, 3-bisphosphoglycerate to 3-phosphoglycerate by phosphoglycerate kinase and in the conversion of phosphoenolpyruvate to pyruvate by pyruvate kinase. Since two molecules of 1,3-bisphosphoglycerate are generated from one glucose molecule (and subsequently two molecules of phosphoenolpyruvate are generated), each of these steps results in generation of two ATPs. Thus, two ATPs are expended in the first step of glycolysis and four ATPs are subsequently generated in later stages, for a net total of two ATPs generated from glycolysis of one molecule of glucose.

Fasting is observed in many religions and occurs with food shortages or fad diets. A man goes on a hunger strike and confines himself to a liquid diet with minimal calories. Which of the following would occur after 4-5 hours? a. Decreased cyclic AMP and increased liver glycogen synthesis b. Increased cyclic AMP and increased liver glycogenolysis c. Decreased epinephrine levels and increased liver glycogenolysis d. Increased Ca2+ in muscle and decreased glycogenolysis e. Decreased Ca2+ in muscle and decreased glycogenolysis

The answer is b.(Murray, pp 153-162. Scriver, pp 1521-1552.)In the presence of low blood glucose, epinephrine or norepinephrine interacts with specific receptors to stimulate adenylate cyclase production of cyclic AMP. Cyclic AMP activates protein kinase, which catalyzes phosphorylation and activation of phosphorylase kinase. Activated phosphorylase kinase activates glycogen phosphorylase, which catalyzes the breakdown of glycogen. Phosphorylase kinase can be activated in two ways. Phosphorylation leads to complete activation of phosphorylase kinase. Alternatively, in muscle, the transient increases in levels of Ca2+ associated with contraction lead to a partial activation of phosphorylase kinase. Ca2+ binds to calmodulin, which is a subunit of phosphorylase kinase. Calmodulin regulates many enzymes in mammalian cells through Ca2+ binding.

Which of the following carbohydrates would be most abundant in the diet of strict vegetarians? a. Amylose b. Lactose c. Cellulose d. Maltose e. Glycogen

The answer is c. (Murray, pp 102-110. Scriver, pp 1521-1552.) Cellulose, the most abundant compound known, is the structural fiber of plants and bacterial walls. It is a polysaccharide consisting of chains of glucose residues linked by β 1→ 4 bonds. Since humans do not have intestinal hydrolases that attack β 1→ 4 linkages, cellulose cannot be digested but forms an important source of "bulk" in the diet. Lactose is a disaccharide of glucose and galactose found in milk. Amylose is an unbranched polymer of glucose residues in α 1→ 4 linkages. Glycogen is a branched polymer of glucose with both α1→ 4 and α1→ 6 linkages. Maltose is a disaccharide of glucose, which is usually the breakdown product of amylose.

Cellulose products are often used as solids (vehicles, binders) in pharmaceutical tablets and to provide texture in food products. They are inert ingredients because animals cannot metabolize cellulose, for which of the following reasons? a. Cellulose is insoluble b. They do not commonly consume cellulose c. They do not have an enzyme to hydrolyze the β linkage d. They do not have an enzyme to hydrolyze the branches

The answer is c. (Murray, pp 102-110.) Glucose (glucopyranose) residues in cellulose are linked by β1 → 4 bonds in straight chains that humans cannot hydrolyze because they do not possess an enzyme to carry out this function. Cellulose is a structural constituent of plants and is strengthened by hydrogen bonds that cross-link the strands. Cellulose is insoluble and is a source of fiber in the diet.

Children with respiratory chain disorders frequently have elevations of citric acid cycle intermediates like succinate or fumarate in their blood. Transfer of H+/e− pairs to electron transport carriers, decarboxylation, and substrate-level phosphorylation occur at some of the steps shown in the following diagram of the citric acid cycle. All three of these events occur at which of the following steps? a. Step A b. Step B c. Step C d. Step D e. Step E

The answer is c. (Murray, pp 123-135. Scriver, pp 2367-2424.)In the citric acid cycle, the conversion of α-ketoglutarate to succinate results in decarboxylation, transfer of an H+/e− pair to NADH+, H+, and the substratelevel phosphorylation of GDP to GTP. The series of reactions involved is quite complex. First, α-ketoglutarate reacts with NAD+ + CoA to yield succinyl-CoA + CO2 + NADH + H+. These reactions occur by the catalysis of the α-ketoglutarate dehydrogenase complex, which contains lipoamide, FAD+, and thiamine pyrophosphate as prosthetic groups. Under the action of succinyl CoA synthetase, succinyl CoA catalyzes the phosphorylation of GDP with inorganic phosphate coupled to the cleavage of the thioester bond of succinyl CoA. Thus, the production of succinate from α-ketoglutarate yields one substrate-level phosphorylation and the production of three ATP equivalents from NADH via oxidative phosphorylation.

Which of the following compounds is recycled in the citric acid cycle and thus serves a catalytic role? a. Acetyl-CoA b. Citrate c. Oxaloacetate d. Succinate

The answer is c. (Murray, pp 130-135. Scriver, pp 2327-2356.) Acetyl-CoA is the entrance substrate to the citric acid cycle. Reaction between acetyl-CoA and oxaloacetate results in production of citrate. Citrate is subsequently metabolized to succinate and then back to oxaloacetate with production of two molecules of CO2. Oxaloacetate is thus regenerated during the cycle and only acts as a catalyst.

McArdle's disease (type V glycogen storage disease, 232600, Table 3) causes muscle cramps and muscle fatigue with increased muscle glycogen. Which of the following enzymes is deficient? a. Hepatic hexokinase b. Muscle glycogen synthetase c. Muscle phosphorylase d. Muscle hexokinase e. Muscle debranching enzyme

The answer is c. (Murray, pp 145-152. Scriver, pp 1521-1552.) Muscle phosphorylase deficiency leads to a glycogen storage disease—McArdle disease (232600)—that in young adults causes inability to do strenuous physical work because of muscular cramps resulting from ischemia. The compromised phosphorylation of muscle glycogen characteristic of McArdle disease compels the muscles to rely on auxiliary energy sources such as free fatty acids and ambient glucose.

After a term uncomplicated gestation, normal delivery, and unremarkable nursery stay, a 10-day-old female is readmitted to the hospital because of poor feeding, weight loss, and rapid heart rate. Antibiotics are started as a precaution against sepsis, and initial testing indicates an unusual echocardiogram with a very short PR interval and a large heart on x-ray. Initial concern about a cardiac arrhythmia changes when a large tongue is noted, causing concern about glycogen storage disease type II (Pompe disease—232300—Table 3). Which of the following best explains why Pompe disease is more severe and lethal compared to other glycogen storage diseases? a. The deficiency is a degradative rather than synthetic enzyme b. The deficiency involves a liver enzyme c. The deficiency involves a lysosomal enzyme d. The deficiency causes associated neutropenia e. The deficiency involves a serum enzyme

The answer is c. (Murray, pp 145-152. Scriver, pp 1521-1552.) Pompe disease has an early and severe onset compared to the other glycogen storage diseases listed in High-Yield Facts Table 4 because the defective α-glucosidase is a lysosomal enzyme. Accumulation of substances in the lysosome often leads to a more severe and progressive course, illustrated by the mucopolysaccharidoses like Hurler syndrome (607014) or the neurolipidoses like Tay-Sachs disease (272800). Patients with Pompe disease exhibit lysosomal glycogen accumulation in muscle (muscle weakness or hypotonia), brain (with developmental retardation), and heart (with a short PR interval and heart failure). The other glycogen storage diseases lead to glycogen accumulation in liver or muscle with correspondingly milder symptoms.

A child presents with low blood glucose (hypoglycemia), enlarged liver (hepatomegaly), and excess fat deposition in the cheeks (cherubic facies). A liver biopsy reveals excess glycogen in hepatocytes. Deficiency of which of the following enzymes best explains this phenotype? a. α-1,1-glucosidase b. α-1,1-galactosidase c. α-1,4-glucosidase d. α-1,4-galactosidase e. α-1,6-galactosidase

The answer is c. (Murray, pp 145-152. Scriver, pp 1521-1552.) The child has symptoms of glycogen storage disease. Glycogen is a glucose polymer with linear regions linked through the C1 aldehyde of one glucose to the C4 alcohol of the next (α-1,4-glucoside linkage). There are also branches from the linear glycogen polymer that have α-1,6-glucoside linkages. Glycogen is synthesized during times of carbohydrate and energy surplus, but must be degraded during fasting to provide energy. Separate enzymes for breakdown include phosphorylases (α-1,4-glucosidases) that cleave linear regions of glycogen and debranching enzymes (α-1,6-glucosidases) that cleave branch points. Glucose-6-phosphatase is needed in the liver to liberate free glucose from glucose-6-phosphate, providing fuel for other organs. There is no glucose-6-phosphatase in muscle, and muscle glycogenolysis provides energy just for muscle with production of lactate. Deficiencies of more than eight enzymes involved in glycogenolysis, including those mentioned, can produce glycogen storage disease.

Which of the following best explains why fructose is often used as a carbohydrate substitute in special foods for patients with diabetes mellitus? a. Fructose is a better substrate for hexokinase b. Fructose stimulates residual insulin release c. Fructose has a specific kinase in liver that allows bypass of phosphofructokinase d. Fructose is phosphorylated and cleaved to triose phosphates, which cannot be used for gluconeogenesis e. Hexokinase phosphorylates fructose in extrahepatic tissues, and its activity will not be affected by high glucose concentrations in diabetes

The answer is c. (Murray, pp 163-172. Scriver, pp 1489-1520.) Three enzymes for fructose metabolism, a specific fructokinase plus aldolase B and triokinase, are present at high levels in liver (also in kidney and small intestine). Fructokinase catalyzes the phosphorylation of fructose to fructose-1phosphate, which is then split to D-glyceraldehyde and dihydroxyacetone by aldolase B. Triokinase converts D-glyceraldehyde to glyceraldehyde3-phosphate, which can be metabolized further by glycolysis or be condensed with dihydroxyacetone phosphate by adolase to form fructose 1, 6-diphosphate, glucose-6-phosphate, and glucose as gluconeogenesis. Foods high in sucrose (glucose-fructose) such as syrups, beverages, or diabetic substitutes yield high concentrations of fructose in the portal vein. Fructose is catabolized more rapidly than glucose by its specific fructokinase, bypassing hexokinase that is regulated by fasting and insulin. It provides a fuel for glycolysis, but also increases fatty acid, VLDL, and cholesterol-LDL production that is not desirable in diabetes mellitus.

A teenage boy presents to clinic complaining of muscle cramps on exercise. Past history indicates he had some coordination problems in childhood and received occupational therapy. Blood tests show an increased amount of lactic acid at rest, with dramatic increases on exercise testing. Simultaneous measures of capillary oxygenation by a surface probe were normal. The abnormality most likely involves which of the following? a. Glycolysis in the lysosomes b. Glycolysis in the cytosol c. Respiratory chain in the mitochondria d. Glycogen breakdown in the mitochondria

The answer is c. (Murray, pp 80-101. Scriver, pp 2261-2296.) Under conditions of plentiful oxygen (aerobic metabolism), pyruvate formed from glycolysis in the cytosol is metabolized to acetylCoA. Acetyl CoA enters the mitochondria and the citric acid cycle in the conversion of oxaloacetate to citrate, generating NADH and FADH2 reducing equivalents that generate ATP through oxidation in the mitochondrial respiratory chain. In respiratory chain disorders, the disrupted electron transport chain does not function as well, causing more pyruvate to be converted to lactate in muscle with muscle weakness and cramping. Lactate from exercising muscle is normally converted to glucose by the liver, but excess lactate produced in severe respiratory chain disorders accumulates in serum to lower the pH (acidosis). Mitochondria are called the powerhouses of the cell since they contain the citric acid cycle and the respiratory chain that generates abundant ATP through oxidative phosphorylation.

A previously normal 2-month-old female is evaluated because of jittery spells several hours after meals. A low blood glucose value is noted, and physical examination demonstrates a liver edge some two finger breadths below the right costal margin. Percussion of the right chest and abdomen confirms that the liver width is slightly enlarged. Hospital testing reveals that the infant can increase her blood glucose after breast feeding but that it is not maintained at normal levels 3-4 hours after feeding. Which of the following is the most likely diagnosis? a. Intestinal malabsorption of lactose b. Galactosemia with inability to convert lactose to glucose c. Fructosemia with inability to liberate sucrose from glucose d. Glycogen storage disease e. Growth hormone deficiency with inability to maintain glucose

The answer is d. (Murray, pp 102-110, 145-152. Scriver, pp 1521-1551.) Important carbohydrates include the disaccharides maltose (glucose-glucose), sucrose (glucose-fructose) and lactose (galactoseglucose), and the glucose polymers starch (cereals, potatoes, vegetables) and glycogen (animal tissues). Humans must convert dietary carbohydrates to simple sugars (mainly glucose) for fuel, employing intestinal enzymes and transport systems for enzymatic digestion and absorption. Simple sugars (galactose, fructose) are converted to glucose by liver enzymes, and the glucose is reversibly stored as glycogen. Enzymatic deficiencies in intestinal digestion (e.g., lactase deficiency in those with lactose intolerance), in sugar to glucose conversion (e.g., galactose to glucose conversion in galactosemia), or in glycogenesis/glycogenolysis (e.g., in those glycogen storage diseases) result in glucose deficiencies (low blood glucose or hypoglycemia) with potential accumulation and toxicity to hepatic tissues. The infant had been normal, excluding low glucose due to growth hormone deficiency, and could readily digest breast milk lactose with absorption and conversion to glucose. Low glucose during fasting and liver enlargement implies altered regulation of glycogen synthesis/release due to one of the enzyme deficiencies within the category of glycogen storage disease.

Which of the following statements about the structure of glycogen is true? a. Glycogen is a copolymer of glucose and galactose b. There are more branch residues than residues in straight chains c. Branch points contain α1φ4 glycosidic linkages d. New glucose molecules are added to the C1 aldehyde group of chain termini, forming a hemiacetal e. The monosaccharide residues alternate between D- and L-glucose

The answer is d. (Murray, pp 102-110. Scriver, pp 1521-1552.) Glycogen is a highly branched polymer of α-D-glucose residues joined by α1→4-glycosidic linkage. Under the influence of glycogen synthase, the C4 alcohol of a new glucose is added to the C1 aldehyde group of the chain terminus. The branched chains occur about every 10 residues and are joined in α1→6-glycosidic linkages. Large amounts of glycogen are stored as 100- to 400-Å granules in the cytoplasm of liver and muscle cells. The enzymes responsible for making or breaking the α1→4-glycosidic bonds are contained within the granules. Thus glycogen is a readily mobilized form of glucose.

The citric acid cycle occurs in which subcellular compartment? a. Cytosol b. Endoplasmic reticulum c. Golgi d. Mitochondria e. Nucleus

The answer is d. (Murray, pp 122-135, 2327-2356.) In addition to the citric acid cycle, mitochondria also house the enzymes for β oxidation of fatty acids and oxidative phosphorylation. The cytosol is the site for glycolysis, glycogenesis, the pentose phosphate pathway, and fatty acid biosynthesis.

Which of the following two compounds are the primary products of the pentose phosphate pathway? a. NAD+ and ribose b. NADH and ribose c. NADP+ and ribose d. NADPH and ribose e. NAD+ and glucose f. NADH and glucose g. NADP+ and glucose h. NADPH and glucose

The answer is d. (Murray, pp 163-172. Scriver, pp 1434-1436.) The pentose phosphate cycle does not produce ATP, but instead produces ribose and NADPH. NADP+ is the hydrogen acceptor instead of NAD+, as in glycolysis. In the oxidative phase of the pentose phosphate pathway, NADPH is generated by glucose-6-phosphate dehydrogenase. NADPH is also generated by 6-phosphogluconate dehydrogenase. Ribose is generated in the nonoxidative phase.

In lung diseases such as emphysema or chronic bronchitis, there is chronic hypoxia that is particularly obvious in vascular tissues such as the lips or nail beds (cyanosis). Certain genetic diseases like α1-antitrypsin deficiency (107400) predispose to emphysema, as do environmental exposures like cigarette smoking or asbestos. Poorly perfused areas exposed to chronic hypoxia have decreased metabolic energy for tissue maintenance and repair. Which of the following is an important reason for this? a. Increased hexokinase activity owing to increased oxidative phosphorylation b. Increased ethanol formation from pyruvate on changing from anaerobic to aerobic metabolism c. Increased glucose utilization via the pentose phosphate pathway on changing from anaerobic to aerobic metabolism d. Decreased ATP generation and increased glucose utilization on changing from aerobic to anaerobic metabolism e. Decreased respiratory quotient on changing from carbohydrate to fat as the major metabolic fuel

The answer is d. (Murray, pp 580-597. Scriver, pp 5559-5586.) The exposure of tissues to chronic hypoxia makes them rely more on anaerobic metabolism for the generation of energy as ATP and other high-energy phosphates. Most tissues except for red blood cells can metabolize glucose under anaerobic or aerobic conditions (red blood cells do not have mitochondria for electron transport and must rely on other tissues to generate glucose back from lactate). In most tissues, a switch from aerobic to anaerobic metabolism greatly increases glucose utilization and decreases energy production. (A reduction of glucose utilization under anaerobic conditions in bacteria is known as the Pasteur effect after its discoverer). Under aerobic conditions, the cell can produce a net gain in moles of ATP formed per mole of glucose utilized that can be as high as 18 times that produced under anaerobic conditions. Thus the cell generates more energy and requires less glucose under aerobic conditions. Such increased ATP concentrations, together with the release of citrate from the citric acid cycle under aerobic conditions, allosterically inhibit the key regulatory enzyme of the glycolytic pathway, phosphofructokinase. Decreased phosphofructokinase activity decreases metabolism of glucose by glycolysis.

A middle-Eastern family presents for evaluation because their infant son died in the nursery with severe hemolysis and jaundice. The couple had two prior female infants who are alive and well, and the wife relates that she lost a brother in infancy with severe hemolysis induced after a viral infection. The physician suspects glucose-6-phosphate dehydrogenase deficiency (305900), implying defective synthesis of which of the following compounds? a. Deoxyribose and NADP b. Glucose and lactate c. Lactose and NADPH d. Ribose and NADPH e. Sucrose and NAD

The answer is d. (Scriver, pp 4517-4554; Murray, pp 163-167.) Glucose-6-phosphate dehydrogenase (G6PD) is the first enzyme of the pentose phosphate pathway, a side pathway for glucose metabolism whose primary purpose is to produce ribose and NADPH. Its deficiency (305900) is the most common enzymopathy, affecting 400 million people worldwide. It contrasts with glycolysis in its use of NADP rather than NAD for oxidation, its production of carbon dioxide, its production of pentoses (ribose, ribulose, xylulose), and its production of the high-energy compound PRPP (5-phosphoribosyl-1-pyrophosphate) rather than ATP. Production of NADPH by the pentose phosphate pathway is crucial for reduction of glutathione, which in turn removes hydrogen peroxide via glutathione peroxidase. Erythrocytes are particularly susceptible to hydrogen peroxide accumulation, which oxidizes red cell membranes and produces hemolysis. Stresses like newborn adjustment, infection, or certain drugs can increase red cell hemolysis in G6PD-deficient individuals, leading to severe anemia, jaundice, plugging of renal tubules with released hemoglobin, renal failure, heart failure, and death. Since the locus encoding G6PD is on the X chromosome, the deficiency exhibits X-linked recessive inheritance with severe affliction in males and transmission through asymptomatic female carriers. Ribose-5-phosphate produced by the pentose phosphate pathway is an important precursor for ribonucleotide synthesis, but alternative routes from fructose-6-phosphate allow ribose synthesis in tissues without the complete cohort of pentose phosphate enzymes or with G6PD deficiency. The complete pentose phosphate pathway is active in liver, adipose tissue, adrenal cortex, thyroid, erythrocytes, testis, and lactating mammary gland. Skeletal muscle has only low levels of some of the enzymes of the pathway but is still able to synthesize ribose through fructose-6-phosphate.

A 7-year-old female presents with dehydration after 3 days of vomiting and diarrhea. Her parents mention that a sibling was diagnosed with a type of diabetes that spilled sugar into the urine but did not need treatment. A urine Clinitest for reducing sugars is strongly positive, but the physician obtains a blood glucose level that is normal; a urine glucose oxidase test on the urine is also negative for glucose. Further analysis of the urine reveals a small amount of fructose and a large amount of an unidentified pentose that is most likely which of the following? a. Galactose b. Glucose c. Lactose d. Mannose e. Xylulose

The answer is e. (Murray, pp 163-172. Scriver, pp 1489-1520.) Before urine test strips were designed with specific enzyme reagents like glucose oxidase, any sugar with a reducing aldehyde or ketone group would reduce the dye and produce a green color reaction. It was therefore important to differentiate glucosuria due to diabetes mellitus or renal tubular problems from other sugars in the urine, like galactose in galactosemia or fructose in essential fructosuria. The uronic acid pathway, like the pentose phosphate pathway, provides an alternate fate for glucose without generating ATP. Glucose-6-phosphate is converted to glucose-1-phosphate and reacted with UTP to form the higher energy compound UDP-glucose. UDP-glucose is converted to UDP-glucuronic acid that is a precursor for glucuronide units in proteoglycan polymers. Unused glucuronic acid is converted to xylulose and then to xylitol by a xylulose reductase, the enzyme deficiency in essential pentosuria (260800). In this "disease," which is better called a trait, excess xylulose is excreted into urine but causes no pathology. Pentoses (5-carbon sugars) are important in the pentose phosphate and uronic acid pathways, providing ribose for nucleic acid metabolism. The other sugars listed as options are all 6-carbon hexoses.

Disorders with abnormalities of the respiratory chain present with central nervous system and muscle symptoms (seizures, low tone) due energy deficiency. A preliminary test to decide if a patient may have a mitochondrial electron transport disorders examines the ratio of pyruvate to that of its product under resting conditions. What is the ratio and how would it be affected by abnormal electron transport? a. Pyruvate/Acetyl-CoA—increased b. Pyruvate/Acetyl-CoA—decreased c. Pyruvate/Glucose—decreased d. Pyruvate/Lactate—increased e. Pyruvate/Lactate—decreased

The answer is e. (Murray, pp 92-101. Scriver, pp 2261-2274. ) Under aerobic conditions, pyruvate is oxidized by pyruvate dehydrogenase to acetyl-CoA, which enters the citric acid cycle. The citric acid cycle generates reducing equivalents in the form of FADH and NADH that are converted to oxygen by the electron transport chain to yield abundant ATP. Under anaerobic conditions such as heavy exercise, pyruvate must be converted to lactate to recycle NADH to NAD+ to allow glycolysis to continue. In mitochondrial disorders resulting from mutations in cytochromes or pyruvate dehydrogenase, there is deficient NADH oxidation and ATP production. Lactate will accumulate as it does normally in tissues without mitochondria (erythrocytes) or in tissues with exercise stress (like muscle). The lactate can accumulate in serum, causing a decreased pyruvate to lactate ratio and lactic acidosis that are typical signs of mitochondrial disease. These abnormalities also occur with circulatory failure (shock) or hypoxemia, so they are suspect for inborn errors only when cardiorespiratory function is normal. Glycolysis produces only 2 ATP compared to the coupling of citric acid intermediates with electron transport that produces 12 ATP per cycle; tissues highly dependent on the respiratory chain (nerves, muscle, retina) are predominantly affected in mitochondrial disorders—for example, Leigh disease. Suggestive signs like the decreased pyruvate/lactate ratio must be followed by more specific tests like muscle biopsy (ragged red fibers), eye examination (retinal pigmentation), or mitochondrial DNA analysis (deletions, point mutations) to diagnose highly variable mitochondrial diseases.

A teenager is brought in by his parents after his physical education teacher gives him a failing grade. The teacher has scolded him for malingering because he drops out of activities after a few minutes of exercise, complaining of leg cramps and fatigue. A stress test is arranged with sampling of blood metabolites and monitoring of exercise performance. Which of the following results after exercise would support a diagnosis of glycogen storage disease in this teenager? a. Increased oxaloacetate, decreased glucose b. Increased glycerol and glucose c. Increased lactate and glucose d. Increased pyruvate and stable glucose e. Stable lactate and glucose

The answer is e. (Scriver, pp 1521-1551. Murray, pp 145-152.) Under circumstances of intense muscular contraction, the rate of formation of NADH by glycolysis exceeds the capacity of mitochondria to reoxidize it. Consequently, pyruvate produced by glycolysis is reduced to lactate, thereby regenerating NAD+. Since erythrocytes have no mitochondria, accumulation of lactate occurs normally. Lactate goes to the liver via the blood, is formed into glucose by gluconeogenesis, and then reenters the bloodstream to be reutilized by erythrocytes or muscle. This recycling of lactate to glucose is called the Cori cycle. A somewhat similar phenomenon using alanine generated by muscles during starvation is called the glucosealanine cycle. All of the other substances listed—oxaloacetate, glycerol, and pyruvate—can be made into glucose by the liver. In muscle, glycogenolysis is synchronized with contraction by epinephrine (through cyclicAMP) and calcium activation of phosphorylase. In those with musclespecific phosphorylase defects (glycogen storage diseases V and VII), glucose is not mobilized as efficiently from glycogen, causing decreased contractile efficiency (cramping, fatigue), decreased yield of lactate from glycolysis, and maintenance of serum glucose by compensating liver metabolism.


Conjuntos de estudio relacionados

Топографія нижнього поверху черевної порожнини

View Set

Cognitive Psychology Exam 2 (Chapters 5-8)

View Set

The first Emperor of china, Ch.22

View Set